Work, Power and Energy — Lifting a coiled rope up off a surface

In summary: Additionally internal energy could always be added as...In Summary, the solution said that Average power= net change in energy/time, but why is that true? If you have a different way of solving it, it would be helpful too.
  • #1
Differentiate it
63
2
Homework Statement
(view the image)
Relevant Equations
W = ∆KE
The solution said that Average power= net change in energy/time, but why is that true? If you have a different way of solving it, it would be helpful too
 

Attachments

  • image.png
    image.png
    32.2 KB · Views: 65
Physics news on Phys.org
  • #2
Differentiate it said:
Homework Statement:: (view the image)
Relevant Equations:: W = ∆KE

The solution said that Average power= net change in energy/time, but why is that true?
That the definition of power: energy per unit time.
 
  • #3
To add to what @PeroK said, 'average' is used because when a quantity Q changes by an amount ΔQ over time-interval Δt, the average rate of change of Q with respect to time during the interval is ΔQ/Δt. That's what average means.

E.g. you travel 10m in 1s, then 30m in 4s. Your average speed during the 5s is (10m+30m)/(1s+4s) = 8m/s.

In your question, you need to ask yourself:
- what is the increase in potential energy?
- what is the increase in kinetic energy?
- what is the time taken, expressed in terms of v and l?
 
  • Like
Likes hutchphd and PeroK
  • #4
Steve4Physics said:
To add to what @PeroK said, 'average' is used because when a quantity Q changes by an amount ΔQ over time-interval Δt, the average rate of change of Q with respect to time during the interval is ΔQ/Δt. That's what average means.

E.g. you travel 10m in 1s, then 30m in 4s. Your average speed during the 5s is (10m+30m)/(1s+4s) = 8m/s.

In your question, you need to ask yourself:
- what is the increase in potential energy?
- what is the increase in kinetic energy?
- what is the time taken, expressed in terms of v and l?
Is the work done by the normal force zero here? If yes,Why?
 
  • #5
Differentiate it said:
Is the work done by the normal force zero here? If yes,Why?
The normal force acts through no distance.
 
  • Like
Likes Differentiate it
  • #6
I'm getting something very close to, but not exactly matching any of those answers...
 
  • #7
erobz said:
I'm getting something very close to, but not exactly matching any of those answers...
Try harder!
 
  • Haha
Likes berkeman
  • #8
PeroK said:
Try harder!
Mind If I PM you what I've done?
 
  • #9
erobz said:
Mind If I PM you what I've done?
Go ahead!
 
  • #10
This is one of a class of rope/chain problems where using conservation of energy and conservation of momentum appear to give conflicting results.
My attitude at one time was that it is safer to trust momentum conservation, but I have come to realise these problems are rather subtle. I now believe the true result depends on detailed properties of the material involved.
The issue is the flexural springiness of the rope. If we treat it as having none, momentum conservation should be the go. At the opposite extreme, energy conservation may be closer, but still not correct because some of the energy will persist in the form of lateral oscillation and/or rotation.
In other instances of these problems, energy conservation can get quite close to the experimental answer. E.g. a rope initially suspended by both ends at one point, then one end released.
The chain fountain is also a member of this class.
 
  • Like
Likes PeroK
  • #11
haruspex said:
This is one of a class of rope/chain problems where using conservation of energy and conservation of momentum appear to give conflicting results.
My attitude at one time was that it is safer to trust momentum conservation, but I have come to realise these problems are rather subtle. I now believe the true result depends on detailed properties of the material involved.
The issue is the flexural springiness of the rope. If we treat it as having none, momentum conservation should be the go. At the opposite extreme, energy conservation may be closer, but still not correct because some of the energy will persist in the form of lateral oscillation and/or rotation.
In other instances of these problems, energy conservation can get quite close to the experimental answer. E.g. a rope initially suspended by both ends at one point, then one end released.
The chain fountain is also a member of this class.
I remember discussing this before in a thread with you (It was someone else HW problem), and I can't find it now. But Yeah, using conservation of energy and conservation of momentum yield different results. I used momentum - its not one of the options, so it wasn't the authors intention, but still...
 
Last edited:
  • #12
erobz said:
I remember discussing this before in a thread with you (It was someone else HW problem), and I can't find it now. But Yeah, using conservation of energy and conservation of momentum yield different result ( I used momentum - its not one of the options).

We might as well talk about this here @PeroK You think applying momentum conservation is unphysical and not actually Newtons formulation and that is the issue behind the differing results? I'm not trying to cause a ruckus but this whole thing is suspicious.
The energy solution is obviously valid. Given the assumption that there is not additional energy in the rope. Additionally internal energy could always be added as an extension of the solution.

The momentum solution involves an unphysical interpretation of Newton's second law and produces a result which cannot be fully analysed in the way the energy solution can.
 
  • #13
PeroK said:
The energy solution is obviously valid. Given the assumption that there is not additional energy in the rope. That could always be added.

The momentum solution involves an unphysical interpretation of Newton's second law and produces a result which cannot be fully analysed in the way the energy solution can.
So basically, you believe energy (not momentum), has the final say. It isn't clear to me why ##dm ~ v ## is unphysical though, but ##m ~dv## is fine.
 
  • #14
erobz said:
So basically, you believe energy (not momentum), has the final say.
It's not a question of the final say. It's a question of verification of the answer. The end state has a given energy and momentum. Unless you postulate additional internal energy there can only be one answer. Not two.

In any case, if you rewrite Newton's second law as ##F = v\frac{dm}{dt}## then you can't complain if you get a seemingly contradictory answer!
 
  • #15
PeroK said:
It's not a question of the final say. It's a question of verification of the answer. The end state has a given energy and momentum. Unless you postulate additional internal energy there can only be one answer. Not two.

In any case, if you rewrite Newton's second law as ##F = v\frac{dm}{dt}## then you can't complain if you get a seemingly contradictory answer!
But that's the whole premise for the derivation of the Rocket Equation.
 
  • #17
Differentiate it said:
Homework Statement:: (view the image)
Relevant Equations:: W = ∆KE

The solution said that Average power= net change in energy/time, but why is that true? If you have a different way of solving it, it would be helpful too
The net change in energy is the net amount of work done by the pulling force between the full coiled position and the full extension of the rope, until it just takes off from the ground (without considering internal forces in the rope explained by @haruspex above).

Both components of work, force and height, are constantly increasing from a zero value up to the maximum force of the weight of the rope off the ground, which should be ##F_{max}=mg=\lambda lg##

Coiled rope.jpg
 
Last edited:
  • #19
erobz said:
So basically, you believe energy (not momentum), has the final say. It isn't clear to me why ##dm ~ v ## is unphysical though, but ##m ~dv## is fine.
Because mass is neither created nor destroyed. It can enter or leave the system under consideration, but it may well bring momentum or take momentum away with it. You then have to account for that gained or lost momentum by regarding it as a virtual force.
E.g., moving cart leaking water. There appears to be no net force on it, but its momentum is not conserved.
 
  • Like
Likes PeroK
  • #20
haruspex said:
Because mass is neither created nor destroyed. It can enter or leave the system under consideration, but it may well bring momentum or take momentum away with it. You then have to account for that gained or lost momentum by regarding it as a virtual force.
E.g., moving cart leaking water. There appears to be no net force on it, but its momentum is not conserved.
I just can't figure out how we calculate the force required to lift the chain at ##v##.

I can reverse engineer it from Work/energy, but how does it follow from Newtons Laws?
 
Last edited:
  • #21
PeroK said:
The energy solution is obviously valid.
Not so fast. If the momentum solution is wrong we need an argument for why. It is relatively easy to find reasons why work might not be conserved.

As I wrote, it depends on the properties of the rope (and the details of the setup).
In this particular problem, we need to assume it is being pulled up from a great height so that we can take the lifted portion as effectively vertical at all times. If rising at speed v then the point of lift-off is also moving at speed v around the coil. Thus, each portion of the rope, once lifted, is rising at speed ##v\sqrt 2## helically.
Further, there may be losses in the flexing and unflexing of the rope as it transitions from horizontal to vertical.

Soapbox: be careful with idealisations; they should be treated as limits. Nothing is completely inelastic nor completely inextensible/incompressible. If we take a spring in general to have a spring constant ##k_s## when strain is being increased (compression or extension) and ##k_r<k_s## when being relaxed then:
  • inelastic is the limit ##k_r\rightarrow 0##.
  • inextensible/incompressible is the limit ##k_s\rightarrow\infty##
  • perfectly elastic is the limit ##k_r/k_s\rightarrow 1##.
In principle, some problems might require one to specify the relative rates at which simultaneous limits are approached.

First, consider a completely floppy, inelastic rope. It takes no work to flex it or stretch it, but no work is recovered after. As each small segment is lifted it undergoes at least a tiny stretch to accelerate quickly from rest to v, or whatever. Momentum conservation would seem to apply.

Next, suppose it has perfect longitudinal elasticity. The stretch that occurs as one segment is lifted can store energy which is then used to assist the rise of the next segment. Its speed may transiently exceed v, explaining the apparent momentum violation. Even so, it does seem likely that some of this energy will persist as vertical oscillations in the rope, not doing any useful work.

Now allow some flexural elasticity. As a segment is lifted, it stores energy in its bending, and the release of that as it leaves the ground can give it an extra vertical impulse.

In short, problems like these cannot be answered, and should not be asked, without quite detailed specification of properties, and even then may be very challenging to solve.
 
  • Like
Likes jbriggs444
  • #22
Another thing. If variable mass is a problem in using conservation of momentum, why is it not an issue with the energies? All of those expressions for energy are derived from a constant mass assumption as well.

For instance the kinetic energy:

$$F = m \frac{dv}{dt} = m \frac{dv}{dx}\frac{dx}{dt} = m \frac{dv}{dx} v $$

The assumption of constant mass ##m## gives:

$$ \int F~ dx = m \int v ~dv$$

The limits of integration don't make any sense for this application. The LHS is identically zero for this problem.

And in this problem the mass ##m## is a function of ##x##, So do we actually have:

$$\int \frac{F(x)}{ \lambda x } ~ dx = 0 $$

?
 
Last edited:
  • #23
erobz said:
Another thing. If variable mass is a problem in using conservation of momentum, why is it not an issue with the energies? All of those expressions for energy are derived from a constant mass assumption as well.

For instance the kinetic energy:

$$F = m \frac{dv}{dt} = m \frac{dv}{dx}\frac{dx}{dt} = m \frac{dv}{dx} v $$

The assumption of constant mass ##m## gives:

$$ \int F~ dx = m \int v ~dv$$

The limits of integration don't make any sense for this application. The LHS is identically zero for this problem.
There's no issue with either momentum or energy so long as you keep track of any that leaves or enters the system along with the mass. In post #17 the system is the entire rope.
 
  • Like
Likes Lnewqban
  • #24
haruspex said:
There's no issue with either momentum or energy so long as you keep track of any that leaves or enters the system along with the mass. In post #17 the system is the entire rope.
We’ll, I’m hearing the impression that because they yield different results there is an issue. Only one should emerge victorious, or they should agree.

Also, I feel like I’ve made a valid point in #22 that wasn’t addressed. The derivations for energy we use are having issues in this application. The mass is actually a function of ##x##. See the edit. Something is going haywire there.
 
  • #25
erobz said:
The LHS is identically zero for this problem.
I don't see why. Please explain what F and x are defined to be in that context.
erobz said:
in this problem the mass m is a function of x,
If you define the system as the portion of rope no longer on the table, that is true. But it is not a problem if we define the table as the zero for potential energy, since that will mean the mass being added adds no energy.
erobz said:
We’ll, I’m hearing the impression that because they yield different results there is an issue. Only one should emerge victorious, or they should agree.
Don’t confuse two issues.

My beef with questions like these is that they are essentially unanswerable without a lot more detail. Without that one cannot know how well work is conserved or whether there might be a bit of kick-up from the table to upset the momentum equation.

But in posts #13 to #16 there was discussed the side issue of using momentum conservation on a variable mass system, as one sometimes sees in treatments of the rocket equation. The popular ##F=\dot p=m\dot v+\dot m v## can lead to the wrong result if carelessly applied.
 
  • #26
haruspex said:
Not so fast. If the momentum solution is wrong we need an argument for why.
@erobz forgot the normal force.

I think we are talking at cross purposes as @erobz messaged me a solution that effectively gives the final KE of the rope as ##mv^2##. There must, therefore, be an error in that calculation.

The error was not in the restatement of Newton's law, but forgetting the normal force.

This is the problem with @erboz hijacking another user's homework.
 
  • #27
PeroK said:
@erobz forgot the normal force.

I think we are talking at cross purposes as @erobz messaged me a solution that effectively gives the final KE of the rope as ##mv^2##. There must, therefore, be an error in that calculation.

The error was not in the restatement of Newton's law, but forgetting the normal force.

This is the problem with @erboz hijacking another user's homework.
Pax.
 
  • #28
PeroK said:
This is the problem with @erboz hijacking another user's homework.
I’m always the bad guy.
 
  • #29
haruspex said:
I don't see why. Please explain what F and x are defined to be in that context.
In light of recent revelations, It’s probably better if the chain is understood to be horizontal being pulled on a frictionless surface.

The Integral on the RHS is integrated from ##v## to ##v##, (we are applying the force at a constant velocity) so it is 0.
 
  • Like
Likes Lnewqban
  • #30
Imagine a chain/rope being pulled horizontally out of a loose pile ## x \rightarrow^+## by a force ##F## at constant velocity ##v## (no friction). What does Work Energy give us?

Classically with a constant mass (not a chain coming out of a pile):

$$\int_{0}^{l} F ~ dx = m \int_{v}^{v} v ~dv = \frac{1}{2} m v^2 = 0 $$

The force ##F## is not doing Work. Seems like everything I was taught.

Now on to the chain:

I feel like we should be able to argue that the mass moving at velocity ##v## is actually a function of ##x## i.e. ##m(x) = \lambda x## and incorporate it into the integral on the LHS and perhaps resolve the issue of no work being done by the force ##F##...

$$ \int_{0}^{l} \frac{F(x)}{\lambda x} dx = \int_{v}^{v} v ~dv = 0 $$

We still get no work?

Lets ignore the fact that the mass being pulled at velocity ##v## is a function of ##x## ( as far as integration is concerned) and leave it on the RHS.

$$ \int_{0}^{l} F(x)~dx = \lambda x \int_{v}^{v} v ~dv = 0 $$

Still no work?

I feel like this is a problem because in order to solve the OP via energy methods, we are basically assuming (for this simplified problem and for the OP) seemingly against all interpretations that:

$$ W = \int_{0}^{l} F ~ dx = \frac{1}{2} \lambda l v^2 \neq 0 $$
 
Last edited:
  • #31
To me it seems like to be consistent we always should be writing the following:

$$ W = \int F ~dx = \int \left( m \frac{dv}{dt} + v \frac{dm}{dt} \right) dx $$

Then we can see that for the chain at constant velocity ##v## problem:

$$ W = \int F ~dx = \int \left( \cancel{ m \frac{dv}{dt}} + v \frac{dm}{dt} \right) dx $$

$$ \implies \int F ~dx = \int v \frac{dm}{dt} dx = \int v \frac{dm}{dx} \frac{dx}{dt} dx = \int v \frac{dm}{dx} v dx = v^2 \int_{0}^{l} \lambda ~dx = \lambda l v^2 $$

However, that doesn't really play nicely either. That factor of ##\frac{1}{2}## out front is missing, or should it not actually be there?
 
  • #32
Another thing that is bothering me.

PeroK said:
@erobz forgot the normal force.

The normal force is then doing work that is changing the outcome?

Differentiate it said:
Is the work done by the normal force zero here? If yes,Why?
PeroK said:
The normal force acts through no distance.
 
  • #33
Okay, I think I've got the issue. I divided the rope up into a large number of segments, then looked at what was happening to the rope already moving upwards at ##v## and the next segment being accelerated to ##v##. Here's the issue. The first of these forces acts through a greater distance as the rope is already moving at ##v##. The average speed of the accelerating segment is only ##\frac v 2##.

So, we cannot use ##P = Fv##. We must use ##P = F_1v + F_2\frac v 2##.

If we have an infinite acceleration to ##v##, then that's where the power calculation goes wrong.

With this approach both methods, energy and momentum yield the same answer.
 
  • Like
Likes erobz, Lnewqban and Steve4Physics
  • #34
Could I please get help with this one too?
 

Attachments

  • Screenshot_2022-11-20-22-42-26-96_e2d5b3f32b79de1d45acd1fad96fbb0f.jpg
    Screenshot_2022-11-20-22-42-26-96_e2d5b3f32b79de1d45acd1fad96fbb0f.jpg
    18.2 KB · Views: 52
  • #35
Differentiate it said:
Could I please get help with this one too?
Could finish the last one so everyone can get on the same page with what the answer should be (or is intended to be according to the author)?
 

Similar threads

  • Introductory Physics Homework Help
2
Replies
38
Views
1K
  • Introductory Physics Homework Help
Replies
5
Views
799
  • Introductory Physics Homework Help
Replies
6
Views
238
  • Introductory Physics Homework Help
Replies
7
Views
1K
Replies
1
Views
579
  • Introductory Physics Homework Help
Replies
3
Views
1K
  • Introductory Physics Homework Help
Replies
11
Views
4K
  • Introductory Physics Homework Help
Replies
2
Views
628
Replies
1
Views
1K
  • Introductory Physics Homework Help
Replies
1
Views
1K
Back
Top